Difference between revisions of "2023 AMC 12A Problems/Problem 6"
(First solution on an AMC problem!) |
(Edit because I screwed up some late) |
||
Line 5: | Line 5: | ||
==Solution== | ==Solution== | ||
− | Let <math>A(6+m,2+n)</math> and <math>B(6-m,2-n)</math>, since (6,2) is their midpoint. Thus, we must find 2m. We find two equations due to <math>A,B</math> both lying on the function <math>y=\log_{2}x</math>. The two equations are then <math>\log_{2}(6+m)=2+n</math> and <math>\log_{2}(6-m)=2-n</math>. Now add these two equations to obtain <math>log_{2}(6+m)+log_{2}(6-m)=4</math>. By logarithm rules, we get <math>log_{2}((6+m)(6-m))=4</math>. By taking 2 to the power of both sides (what's the word for this?) we obtain <math>(6+m)(6-m)=16</math>. We then get <cmath>36-m^2=16 \rightarrow m^2=20 \rightarrow m=2\sqrt{5}</cmath>. Since we're looking for <math>2m</math>, we obtain <math>2*2\sqrt{5}=\boxed{\textbf{(D) }4\sqrt{5}}</math> | + | Let <math>A(6+m,2+n)</math> and <math>B(6-m,2-n)</math>, since <math>(6,2)</math> is their midpoint. Thus, we must find <math>2m</math>. We find two equations due to <math>A,B</math> both lying on the function <math>y=\log_{2}x</math>. The two equations are then <math>\log_{2}(6+m)=2+n</math> and <math>\log_{2}(6-m)=2-n</math>. Now add these two equations to obtain <math>\log_{2}(6+m)+\log_{2}(6-m)=4</math>. By logarithm rules, we get <math>\log_{2}((6+m)(6-m))=4</math>. By taking 2 to the power of both sides (what's the word for this?) we obtain <math>(6+m)(6-m)=16</math>. We then get <cmath>36-m^2=16 \rightarrow m^2=20 \rightarrow m=2\sqrt{5}</cmath>. Since we're looking for <math>2m</math>, we obtain <math>2*2\sqrt{5}=\boxed{\textbf{(D) }4\sqrt{5}}</math> |
~amcrunner (yay, my first AMC solution) | ~amcrunner (yay, my first AMC solution) |
Revision as of 23:06, 9 November 2023
Problem
Points and lie on the graph of . The midpoint of is . What is the positive difference between the -coordinates of and ?
Solution
Let and , since is their midpoint. Thus, we must find . We find two equations due to both lying on the function . The two equations are then and . Now add these two equations to obtain . By logarithm rules, we get . By taking 2 to the power of both sides (what's the word for this?) we obtain . We then get . Since we're looking for , we obtain
~amcrunner (yay, my first AMC solution)
See Also
2023 AMC 12A (Problems • Answer Key • Resources) | |
Preceded by Problem 5 |
Followed by Problem 7 |
1 • 2 • 3 • 4 • 5 • 6 • 7 • 8 • 9 • 10 • 11 • 12 • 13 • 14 • 15 • 16 • 17 • 18 • 19 • 20 • 21 • 22 • 23 • 24 • 25 | |
All AMC 12 Problems and Solutions |
The problems on this page are copyrighted by the Mathematical Association of America's American Mathematics Competitions.